This is a resolve reconcile explain question, as the question stem asks: Which one of the following, if true, is most helpful in resolving the apparent paradox?

The stimulus opens with a study by the government, which discovered that consumers who opted for bottled water were receiving a more expensive and dangerous product than the public water supply. Seems like a bad deal! Weirdly, even though the study received a lot of attention, people have been buying even more bottled water. The question stem asks us to resolve a paradox; in this case the paradox is that the opposite outcome we would have expected followed the release of the study. Our job is to select the answer choice which provides the best hypothesis for why this weird result occurred. Let’s see what we get:

Answer Choice (A) This seems like another reason for why people should be choosing drinking water instead of bottled water, and does nothing to explain the fact that they aren’t.

Answer Choice (B) This eliminates a potential difference between the two water sources which might explain the consumer preference, and therefore makes the result even weirder without explaining it at all.

Correct Answer Choice (C) This answer explains the paradox by differentiating between subsets of bottled water. Although many kinds of bottled water were less safe than public water, the increase in sales actually came from specifically the brands that were identified as superior to public water in the report.

Answer Choice (D) This makes it slightly less weird, but still does nothing to explain why there was an increase at all.

Answer Choice (E) We are talking about bottled water, not food, and regardless this suggests that consumers do care about what the government says, which makes their apparent disregard of the study stranger.


Comment on this

This is a resolve, reconcile, or explain question, since the stem asks us: Which one of the following, if true, most helps to explain the study’s finding?

The stimulus begins with a yearlong study, in which half of the participants were given a kit to test their cholesterol. The result of the study was that the participants with testing kits reduced, on average, their cholesterol levels 15% more than the no-kit group. Interesting! So being able to test your cholesterol levels seems to help reduce them. The stimulus finishes with two final tidbits; the participants were (i) randomly selected, and specifically from (ii) those who have very high cholesterol levels. Our job here is to explain why having access to testing kits would enable high-cholesterol individuals to reduce their cholesterol more. We are looking for a powerful hypothesis for the difference between the kit-havers and kit-not-havers which relies on as few assumptions as possible. Let’s see what we get:

Answer Choice (A) All participants had high cholesterol, and it’s unclear how accuracy would even affect the amount participants would reduce their cholesterol.

Answer Choice (B) This would make things even more confusing, because they would have lowered their cholesterol by more while avoiding cholesterol-lowering food!

Answer Choice (C) Ok? This gives us no reason for why the tests themselves helped people lower their cholesterol levels.

Answer Choice (D) Our phenomenon is a difference between the kit-havers and kit-not-havers; this information does nothing to explain that difference among the subsets.

Correct Answer Choice (E) If using the kit reinforced cholesterol reduction efforts, then it would make sense why kit-havers reduced their cholesterol by a greater amount!


Comment on this

This page shows a recording of a live class. We're working hard to create our standard, concise explanation videos for the questions in this PrepTest. Thank you for your patience!

This is a strengthening question, as we are asked which answer choice: if true, provides the most support for the argument?

The stimulus begins by informing us that there are two subsets of a particular species of bird; crested and non-crested. The birds usually live in flocks composed of only one of these subsets, and have a preference for birds of the same variety as themselves. So in a crested flock you are likely to find only crested birds, and these birds will likely select crested mates. Interesting! We are further told that even if you move a bird from a flock where all the other birds are crested into a mixed flock, it will select a crested mate regardless of whether it itself is crested or non-crested. It’s important in this sentence that we realize that a flock where all other birds are crested does not mean the bird in question is itself crested. So it seems like non-crested birds from crested flocks will go against the general trend of the birds selecting mates of the same type. The author concludes from this that the preference for mates is learned rather than genetically determined.

If we think about the author’s reasoning, it does make some sense. We would expect birds to mostly select the same kind of mate if preference was learned since flocks are usually composed of one type, and this would also explain why there are exceptions when a bird is raised in a flock mostly of the other type. Since we are choosing one hypothesis (learned) over another (genetic) we should look for answers that support the learned hypothesis over the genetic one. Let’s look at our options:

Answer Choice (A) OK? First of all we are not interested in other bird species, we are interested in why this particular species has a pattern in its preferences for specifically whether a mate is crested or non-crested.

Answer Choice (B) Interesting! But we want support for why a particular behavior (mate selection) is caused by nurture rather than nature.

Answer Choice (C) We want to know why they have a particular preference, not whether they have other preferences.

Answer Choice (D) This actually weakens our learned trait hypothesis, because it shows that a bird can have a mating preference if there was no opportunity to have learned it since the bird was raised in captivity.

Correct Answer Choice (E) This strengthens our learned hypothesis by showing that in the absence of a crested or non-crested homogenous flock, birds lack the mating preference. This is what we would expect if the preference was learned, and not expect if it was genetic.


Comment on this

This page shows a recording of a live class. We're working hard to create our standard, concise explanation videos for the questions in this PrepTest. Thank you for your patience!

This is a very standard NA question stem. If the argument depends on an assumption, the assumption is necessary.

So the plastic rings on a six-pack ensnare and suffocate animals. Yeah, I’ve definitely heard this and always try to cut these up before discarding. I don’t know if it actually helps, but can’t hurt. Anyway, what about it? Oh okay, new rings that will disintegrate after three days of sunlight. A few things about this sentence. First, is this going to work? I mean, it seems like these might get this much light just in shipping and handling before their job is done. This does not matter though. Despite any potential downside, this statement tells us that ALL beverage companies will soon be using these rings. That seems like a pretty big shift. Good. And once we complete the switch to the new rings, the threat plastic rings pose to wildlife will be eliminated. So this is the conclusion.

I can see a few possible problems here, but we don’t want to go hunting for anything specific. For NA questions, it is best to go into the answer choices with an open mind. Let them speak to you and consider the issues they suggest.

Answer Choice (A) Does this have to be true for the argument to work? No. What if some of them will disintegrate in only two days? Well that might be a problem as far as these rings efficacy as packaging, but our conclusion is only about eliminating threat to wildlife. If three days is good, two seems like it would be even better. So when we negate this, it seems to only make things better for our wildlife friends.

Answer Choice (B) We don’t care about this at all. What matters is that these companies are making the switch. End of story. This can bankrupt them for all we care.

Correct Answer Choice (C) So if this isn’t true, has the threat been eliminated? I would have to say it has not. Three days seems like fast disintegration time, but now that this answer directs my attention to it, the conclusion is quite strong. Our argument is claiming to eliminate the threat. Elimination is as absolute as it gets. But these things have up to three days to be out there before they fall apart. And that’s enough time to harm some wildlife. So now it seems the threat is not eliminated without this answer choice. The argument does, indeed, require this to be true. So this is our answer.

Answer Choice (D) Interesting suggestion, but this is not necessary. If we negate this, we’re only expanding the harms being caused by the old rings. But the conclusion is limited to eliminating the threat of suffocation—not all possible threats—so even if there are other threats, they are not relevant to our argument.

Answer Choice (E) Well I certainly hope not. Regardless, does this have to be true? No. We care about the threat of suffocation which remains a threat even if some animals are able to escape the rings. A threat need not be universally fatal to qualify as a threat. A 99% mortality rate would still be quite threatening.

Answers D and E may be tempting because they address the old rings. There is no reason to believe that the old rings will immediately disappear from the environment. They may continue posing a threat even after their use is discontinued in new packaging. So there is an additional necessary condition for this argument which concerns eliminating the risk posed by the old rings, but this answer does not express that assumption correctly. This alternative necessary assumption is a likely pre-phrase, and the possible existence of multiple necessary assumptions is why this strategy is not recommended for the question type. If you D or E based on a pre-phrase, ask yourself: Did you really scrutinize answer choice C to consider what it was suggesting? Or did you dismiss it because it didn’t match your expectation?


Comment on this

This is a pseudo-sufficient assumption question because of the stem: Which one of the following principles... supports the view that a physician’s responding to the request would violate medical ethics? This is at once a PSAr question and a

The stimulus tells us that the police have a wanted poster in a medical journal for a specific fugitive with a non-infectious skin condition that would eventually require him to visit a doctor, asking for information on where the fugitive is. Because doctors are required to report gunshot wounds to police and certain infectious disease to the health authorities, reporting information about this fugitive, according to the argument, would not violate medical ethics. Our author concludes that these exceptions to confidentiality, meaning sharing information on our fugitive, are clearly ethical.

if we read those requirements again very closely, we’ll see that the conclusion does not follow logically from this. Based on the requirement (report to police if it’s a gunshot wound OR health authorities if it’s certain types of infectious diseases), the physician in our stimulus shouldn’t report our fugitive since he has neither a gunshot wound nor an infectious disease.

Figuring out what the question stem is asking us to do is extremely important here. Since our question stem is essentially asking us to support a view that disagrees with the stimulus, our correct answer choice will weaken the argument that the doctor is not violating medical ethics. In other words, we’re trying to weaken the idea that a doctor reporting to the authorities with information on the fugitive is an ethical exception to confidentiality.

Answer Choice (A) This rule brings forward requirements as citizens to report certain issues to the authorities, and nothing about the doctor violating medical ethics. This doesn’t weaken our conclusion by way of a rule/assumption.

Answer Choice (B) We still have to observe the requirements put forward in the stimulus; this answer choice would go against our requirements.

Correct Answer Choice (C) This is correct because it’s pointing out that unless the fugitive had a gunshot wound (thereby requiring the doctor to report the fugitive to the police), the doctor should not be sharing information on the fugitive with anyone (which would then reduce their willingness to come in to be treated for fear of being detained).

Answer Choice (D) Similar to answer choice (B), this would go against the requirement laid out in the stimulus.

Answer Choice (E) Again, we are given exceptions to confidentiality (gunshot wounds and infectious diseases), so this answer choice goes against this requirement, much like answer choices B and D do.


2 comments

This page shows a recording of a live class. We're working hard to create our standard, concise explanation videos for the questions in this PrepTest. Thank you for your patience!

This is a sufficient assumption question, as indicated by the question stem: The conclusion above is properly drawn if which one of the following is assumed?

This is a very conditional-heavy stimulus.

Premise 1: /strike → increase wages

Premise 2: increase wages → sell sub

Conclusion: sell sub

Remember, conditionals have to be triggered in order for us to conclude anything from them. Since we’re trying to prove that Bell’s subsidiaries will be sold, we need something to trigger the first or second premise: either that there will not be a strike or that the wages will be increased.

Answer Choice (A) This does have anything to do with our premise or conclusion and it does help validate our conclusion. It doesn’t trigger our conditionals either.

Answer Choice (B) This is negating the necessary condition in the first premise, which would trigger backward, allowing us to conclude that the workers will do on strike. This answer choice also negates the sufficient condition in premise 2, which renders the rule useless. This is incorrect.

Correct Answer Choice (C) This triggers the first premise, which also triggers our second premise, leading to the conclusion that Bell’s subsidiaries will be sold.

Answer Choice (D) This AC is saying the president can increase wages; this does not mean that he will.

Answer Choice (E) This is additional information that does not clearly trigger our conditionals. The workers only wanting a wage increase does not help us validate the conclusion.


Comment on this

This is a must be true question, as the question stem demands: If the statements above are all true, which one of the following must also be true*?*

The first sentence tells us that Angela will earn her Psychology degree once she completes two courses; one in experimental design, the other in developmental psychology. Unfortunately for Angela, experimental design is a pre-requisite of developmental psychology, and won’t be available till the following term, so Angela won’t finish her degree for at least two more terms. The first sentence tells us that the two courses are enough (i.e. sufficient) for Angela to get her degree. We should infer from the second sentence that they are also required (i.e. necessary) for Angela to get her degree, and that ED is required for DP. It’s important for us to recognize that Angela getting a psychology degree and Angela completing developmental psychology are both sufficient and necessary for each other, and that what we have is therefore a biconditional relationship between the two events. If we were to translate these requirements into a conditional chain, it would be something like: P Degree ←→ DP → ED. Since this is all the information the stimulus has for us, we should expect the correct answer to be something guaranteed by these combined conditionals. Let’s see what the answer choices have in store for us:

Answer Choice (A) We’ve been told nothing about how long the course will take.

Answer Choice (B) We’ve been told nothing about the relative difficulty of the two courses.

Answer Choice (C) We’ve been told nothing about the pre-requisites for experimental design, just that it is required for Angela’s degree and to take developmental psychology.

Answer Choice (D) We’ve been told about the requirements for Angela to get a degree, but we can’t infer that is true of anyone seeking a psychology degree in her university. Maybe there are several options to receive the degree, but Angela’s course selection has committed her to needing these two credits specifically.

Correct Answer Choice (E) Angela’s degree needs both DP and ED, but since DP already requires that she have completed ED, then she must have satisfied all the requirements for her degree if she completes DP.


Comment on this

This is a must be true question, since the stem asks: If the claims above are true, which one of the following must, on the basis of them, be true?

The first sentence begins by informing us that planetary bodies vary in what they’re composed of, but most of those in the Solar System have solid surfaces. It may be helpful to think of this sentence in terms of a superset, planetary bodies, and two overlapping subsets, planetary bodies in the solar system and planetary bodies with solid surfaces. The next sentence begins with the conditional indicator unless, and tells us that the renewal of the surface of a planetary body with a solid surface requires enough heat for volcanic activity. This is followed up with another conditional sentence, beginning with the indicator any, that tells that a solid surface planetary body that does not renew its surface will eventually become covered in craters like the moon. The stimulus ends by telling us some of the specifically old planetary bodies in the Solar System, such as Europa, have solid icy surfaces with very few meteorite craters. Interesting!

When we facing a MBT question with lots of conditionals and modifiers such as this one, its important to get a sense of the logic behind the text. We should look for how our conditionals can connect and relate to each other. In this case what we should notice is that if a planetary body doesn’t have a hot enough core, then it must be heavily marked by craters. Since we know some planetary bodies with solid surfaces are icy and not heavily marked by craters, then it must be true that some planetary bodies with icy surfaces have hot enough cores for volcanic activity. Let’s see if any of the answer choices resemble this pre-phrase:

Answer Choice (A) This is a good example of an answer choice where it is important to remember not to bring in outside knowledge to the LSAT. I’m sure many of you know that the moon isn’t icy, but it is still false that this must be true just based off our stimulus.

Answer Choice (B) Remember our different supersets and subsets. We’ve only been told that the renewal requirement applies to specifically solid surface planetary bodies, but this answer choice applies it to planetary bodies as a whole. It’s important to have picked up on the “such a planetary body” referential phrasing in the second sentence to avoid falling for this answer.

Answer Choice (C) We’ve been told nothing about the proportion of solid surface planetary bodies without lots of craters that are icy. For all we know they are all icy.

Answer Choice (D) We are only told about one of Jupiter’s moons, and that it is not heavily pockmarked.

Correct Answer Choice (E) This is exactly what we pre-phrased. If the only way for a solid surface planetary body to not be heavily marked by craters is to be hot enough for volcanic activity, and we’ve been told about a very cold solid surface planetary body that is not heavily marked by crates, then there must be at least some (i.e Europa) very cold planetary bodies with hot enough cores for volcanic activity.

This page shows a recording of a live class. We're working hard to create our standard, concise explanation videos for the questions in this PrepTest. Thank you for your patience!

Comment on this

This is a pseudo-sufficient assumption question, as the question stem indicates: Which one of the following principles, if established, would most help to justify a doctor’s decision to use drug Z rather than drug Y when treating a patient? Then question stem gives us the information that we need to justify. Let’s see how this appears in the stimulus.

We have information on two studies conducted for drugs Y and Z to prevent blood clots from developing after a heart attack. Y is not more effective than Z (which costs more); but Z could be slightly more efficient or is just as effective as Y. The people who made Z don’t contest these claims but they add that the study does not reveal the added benefits of Z. We’re then told because it’s not clear that Z is more effective than Y, there is no medical reason to prescribe it over Y. Our question stem is asking us to pick the answer that most justifies the reasoning of a doctor who wants to use drug Z (more expensive and potentially slight more effective) over drug Y. A general rule would take evidence from the argument (have it more readily in stock or could be more effective) and put take in the sufficient condition in a conditional with our conclusion in the necessary condition: If they have it in stock/it could be more effective, doctors should prescribe it. Remember that PSAr answer choices do not need to render the argument valid, just somewhere close to it.

Answer Choice (A) doesn’t justify why a doctor can prescribe Z over Y, it justifies a different situation of what requirements patients have to meet in order to be given a drug. This does not justify.

Answer Choice (B) is also incorrect because we’re not concerned with compensation. Deserving something doesn’t justify their choice of Z over Y.

Answer Choice (C) tells us what we should not take into account but fails to mention how we can justify choosing drug Z over drug Y. This is also out.

Answer Choice (D) doesn’t apply to our situation because the drug company doesn’t criticize the findings! It just adds that the findings failed to mention the benefits.

Correct Answer Choice (E): This answer choice is correct; it takes information from the stimulus (potentially more beneficial) and uses it to justify choosing Z over Y.


Comment on this